Trying to prove $e$'s irrationality











up vote
3
down vote

favorite












Knowing that $limlimits_{xto 0} $$frac{sin(x)}{x}$$= 1$ , $frac{1}{n+1}<n!r_n<frac{1}{n}$, where $r_n=e- sum _{ k=0 }^{ n}{ frac { 1 }{k!}} $



By studying $limlimits_{ntoinfty} nsin(2πn!r_n)$ I have to show that :



$$limlimits_{ntoinfty} nsin(2πn!e)= 2π,$$
and then prove that $e$ is irrational ?










share|cite|improve this question
























  • Well, ok. The first limit seems manageable. What are your thoughts?
    – Yuriy S
    Nov 18 at 12:17










  • I tried by composition because $limlimits_{ntoinfty} 2πn!r_n = 0$ and I showed $$limlimits_{ntoinfty} nsin(2πn!r_n)= 2π$$ but i don't think that's correct
    – Lamethyste
    Nov 18 at 12:26












  • Seems fine to me. But not sure about the other limit
    – Yuriy S
    Nov 18 at 12:48










  • Shouldn't the sum in the definition of $r_n$ start at $k=0$ instead of $k=1$?
    – Barry Cipra
    Nov 18 at 12:50










  • You are right, my bad!
    – Lamethyste
    Nov 18 at 12:55















up vote
3
down vote

favorite












Knowing that $limlimits_{xto 0} $$frac{sin(x)}{x}$$= 1$ , $frac{1}{n+1}<n!r_n<frac{1}{n}$, where $r_n=e- sum _{ k=0 }^{ n}{ frac { 1 }{k!}} $



By studying $limlimits_{ntoinfty} nsin(2πn!r_n)$ I have to show that :



$$limlimits_{ntoinfty} nsin(2πn!e)= 2π,$$
and then prove that $e$ is irrational ?










share|cite|improve this question
























  • Well, ok. The first limit seems manageable. What are your thoughts?
    – Yuriy S
    Nov 18 at 12:17










  • I tried by composition because $limlimits_{ntoinfty} 2πn!r_n = 0$ and I showed $$limlimits_{ntoinfty} nsin(2πn!r_n)= 2π$$ but i don't think that's correct
    – Lamethyste
    Nov 18 at 12:26












  • Seems fine to me. But not sure about the other limit
    – Yuriy S
    Nov 18 at 12:48










  • Shouldn't the sum in the definition of $r_n$ start at $k=0$ instead of $k=1$?
    – Barry Cipra
    Nov 18 at 12:50










  • You are right, my bad!
    – Lamethyste
    Nov 18 at 12:55













up vote
3
down vote

favorite









up vote
3
down vote

favorite











Knowing that $limlimits_{xto 0} $$frac{sin(x)}{x}$$= 1$ , $frac{1}{n+1}<n!r_n<frac{1}{n}$, where $r_n=e- sum _{ k=0 }^{ n}{ frac { 1 }{k!}} $



By studying $limlimits_{ntoinfty} nsin(2πn!r_n)$ I have to show that :



$$limlimits_{ntoinfty} nsin(2πn!e)= 2π,$$
and then prove that $e$ is irrational ?










share|cite|improve this question















Knowing that $limlimits_{xto 0} $$frac{sin(x)}{x}$$= 1$ , $frac{1}{n+1}<n!r_n<frac{1}{n}$, where $r_n=e- sum _{ k=0 }^{ n}{ frac { 1 }{k!}} $



By studying $limlimits_{ntoinfty} nsin(2πn!r_n)$ I have to show that :



$$limlimits_{ntoinfty} nsin(2πn!e)= 2π,$$
and then prove that $e$ is irrational ?







limits exponential-function irrational-numbers






share|cite|improve this question















share|cite|improve this question













share|cite|improve this question




share|cite|improve this question








edited Nov 18 at 12:55

























asked Nov 18 at 11:58









Lamethyste

424




424












  • Well, ok. The first limit seems manageable. What are your thoughts?
    – Yuriy S
    Nov 18 at 12:17










  • I tried by composition because $limlimits_{ntoinfty} 2πn!r_n = 0$ and I showed $$limlimits_{ntoinfty} nsin(2πn!r_n)= 2π$$ but i don't think that's correct
    – Lamethyste
    Nov 18 at 12:26












  • Seems fine to me. But not sure about the other limit
    – Yuriy S
    Nov 18 at 12:48










  • Shouldn't the sum in the definition of $r_n$ start at $k=0$ instead of $k=1$?
    – Barry Cipra
    Nov 18 at 12:50










  • You are right, my bad!
    – Lamethyste
    Nov 18 at 12:55


















  • Well, ok. The first limit seems manageable. What are your thoughts?
    – Yuriy S
    Nov 18 at 12:17










  • I tried by composition because $limlimits_{ntoinfty} 2πn!r_n = 0$ and I showed $$limlimits_{ntoinfty} nsin(2πn!r_n)= 2π$$ but i don't think that's correct
    – Lamethyste
    Nov 18 at 12:26












  • Seems fine to me. But not sure about the other limit
    – Yuriy S
    Nov 18 at 12:48










  • Shouldn't the sum in the definition of $r_n$ start at $k=0$ instead of $k=1$?
    – Barry Cipra
    Nov 18 at 12:50










  • You are right, my bad!
    – Lamethyste
    Nov 18 at 12:55
















Well, ok. The first limit seems manageable. What are your thoughts?
– Yuriy S
Nov 18 at 12:17




Well, ok. The first limit seems manageable. What are your thoughts?
– Yuriy S
Nov 18 at 12:17












I tried by composition because $limlimits_{ntoinfty} 2πn!r_n = 0$ and I showed $$limlimits_{ntoinfty} nsin(2πn!r_n)= 2π$$ but i don't think that's correct
– Lamethyste
Nov 18 at 12:26






I tried by composition because $limlimits_{ntoinfty} 2πn!r_n = 0$ and I showed $$limlimits_{ntoinfty} nsin(2πn!r_n)= 2π$$ but i don't think that's correct
– Lamethyste
Nov 18 at 12:26














Seems fine to me. But not sure about the other limit
– Yuriy S
Nov 18 at 12:48




Seems fine to me. But not sure about the other limit
– Yuriy S
Nov 18 at 12:48












Shouldn't the sum in the definition of $r_n$ start at $k=0$ instead of $k=1$?
– Barry Cipra
Nov 18 at 12:50




Shouldn't the sum in the definition of $r_n$ start at $k=0$ instead of $k=1$?
– Barry Cipra
Nov 18 at 12:50












You are right, my bad!
– Lamethyste
Nov 18 at 12:55




You are right, my bad!
– Lamethyste
Nov 18 at 12:55










1 Answer
1






active

oldest

votes

















up vote
1
down vote



accepted










Let $I_n=sumlimits_{k=0}^ndfrac{n!}{k!}in Bbb Z$, then
$$nsin(2pi n! e)=nsin(2pi I_n+2pi n!r_n)=nsin(2pi n!r_n).$$
Note
$$frac 1{n+1}le n!r_n=frac 1{n+1}+frac 1{(n+1)(n+2)}+cdotslesum_{k=1}^inftyfrac1{(n+1)^k}=frac 1n.$$
By sandwich theorem, $n!r_nto 0$ and $ncdot n!r_nto 1$ as $ntoinfty$. Using $sinthetasimtheta$ as $thetato 0$, we get
$$lim_{ntoinfty}nsin(2pi n! e)=lim_{ntoinfty}nsin(2pi n! r_n)=2pi.$$





To prove $e$ irrational, assume $e=p/q$, then when $n>q$, $n!einBbb Z$ implies $sin(2pi n! e)equiv 0$, and
$$lim_{ntoinfty}nsin(2pi n! e)=lim_{ntoinfty}ncdot 0=0ne 2pi,$$
a contradiction.






share|cite|improve this answer























  • I love you :), you made it clear, thanks!
    – Lamethyste
    Nov 18 at 13:07












  • The key idea here is the bound for $r_n$. If $e$ were rational say $m/n$ then $n! e=I_n+n! r_n$ is an integer and this is a contradiction as $n! r_nin[1/(n+1),1/n] $.
    – Paramanand Singh
    Nov 18 at 14:07













Your Answer





StackExchange.ifUsing("editor", function () {
return StackExchange.using("mathjaxEditing", function () {
StackExchange.MarkdownEditor.creationCallbacks.add(function (editor, postfix) {
StackExchange.mathjaxEditing.prepareWmdForMathJax(editor, postfix, [["$", "$"], ["\\(","\\)"]]);
});
});
}, "mathjax-editing");

StackExchange.ready(function() {
var channelOptions = {
tags: "".split(" "),
id: "69"
};
initTagRenderer("".split(" "), "".split(" "), channelOptions);

StackExchange.using("externalEditor", function() {
// Have to fire editor after snippets, if snippets enabled
if (StackExchange.settings.snippets.snippetsEnabled) {
StackExchange.using("snippets", function() {
createEditor();
});
}
else {
createEditor();
}
});

function createEditor() {
StackExchange.prepareEditor({
heartbeatType: 'answer',
convertImagesToLinks: true,
noModals: true,
showLowRepImageUploadWarning: true,
reputationToPostImages: 10,
bindNavPrevention: true,
postfix: "",
imageUploader: {
brandingHtml: "Powered by u003ca class="icon-imgur-white" href="https://imgur.com/"u003eu003c/au003e",
contentPolicyHtml: "User contributions licensed under u003ca href="https://creativecommons.org/licenses/by-sa/3.0/"u003ecc by-sa 3.0 with attribution requiredu003c/au003e u003ca href="https://stackoverflow.com/legal/content-policy"u003e(content policy)u003c/au003e",
allowUrls: true
},
noCode: true, onDemand: true,
discardSelector: ".discard-answer"
,immediatelyShowMarkdownHelp:true
});


}
});














 

draft saved


draft discarded


















StackExchange.ready(
function () {
StackExchange.openid.initPostLogin('.new-post-login', 'https%3a%2f%2fmath.stackexchange.com%2fquestions%2f3003449%2ftrying-to-prove-es-irrationality%23new-answer', 'question_page');
}
);

Post as a guest















Required, but never shown

























1 Answer
1






active

oldest

votes








1 Answer
1






active

oldest

votes









active

oldest

votes






active

oldest

votes








up vote
1
down vote



accepted










Let $I_n=sumlimits_{k=0}^ndfrac{n!}{k!}in Bbb Z$, then
$$nsin(2pi n! e)=nsin(2pi I_n+2pi n!r_n)=nsin(2pi n!r_n).$$
Note
$$frac 1{n+1}le n!r_n=frac 1{n+1}+frac 1{(n+1)(n+2)}+cdotslesum_{k=1}^inftyfrac1{(n+1)^k}=frac 1n.$$
By sandwich theorem, $n!r_nto 0$ and $ncdot n!r_nto 1$ as $ntoinfty$. Using $sinthetasimtheta$ as $thetato 0$, we get
$$lim_{ntoinfty}nsin(2pi n! e)=lim_{ntoinfty}nsin(2pi n! r_n)=2pi.$$





To prove $e$ irrational, assume $e=p/q$, then when $n>q$, $n!einBbb Z$ implies $sin(2pi n! e)equiv 0$, and
$$lim_{ntoinfty}nsin(2pi n! e)=lim_{ntoinfty}ncdot 0=0ne 2pi,$$
a contradiction.






share|cite|improve this answer























  • I love you :), you made it clear, thanks!
    – Lamethyste
    Nov 18 at 13:07












  • The key idea here is the bound for $r_n$. If $e$ were rational say $m/n$ then $n! e=I_n+n! r_n$ is an integer and this is a contradiction as $n! r_nin[1/(n+1),1/n] $.
    – Paramanand Singh
    Nov 18 at 14:07

















up vote
1
down vote



accepted










Let $I_n=sumlimits_{k=0}^ndfrac{n!}{k!}in Bbb Z$, then
$$nsin(2pi n! e)=nsin(2pi I_n+2pi n!r_n)=nsin(2pi n!r_n).$$
Note
$$frac 1{n+1}le n!r_n=frac 1{n+1}+frac 1{(n+1)(n+2)}+cdotslesum_{k=1}^inftyfrac1{(n+1)^k}=frac 1n.$$
By sandwich theorem, $n!r_nto 0$ and $ncdot n!r_nto 1$ as $ntoinfty$. Using $sinthetasimtheta$ as $thetato 0$, we get
$$lim_{ntoinfty}nsin(2pi n! e)=lim_{ntoinfty}nsin(2pi n! r_n)=2pi.$$





To prove $e$ irrational, assume $e=p/q$, then when $n>q$, $n!einBbb Z$ implies $sin(2pi n! e)equiv 0$, and
$$lim_{ntoinfty}nsin(2pi n! e)=lim_{ntoinfty}ncdot 0=0ne 2pi,$$
a contradiction.






share|cite|improve this answer























  • I love you :), you made it clear, thanks!
    – Lamethyste
    Nov 18 at 13:07












  • The key idea here is the bound for $r_n$. If $e$ were rational say $m/n$ then $n! e=I_n+n! r_n$ is an integer and this is a contradiction as $n! r_nin[1/(n+1),1/n] $.
    – Paramanand Singh
    Nov 18 at 14:07















up vote
1
down vote



accepted







up vote
1
down vote



accepted






Let $I_n=sumlimits_{k=0}^ndfrac{n!}{k!}in Bbb Z$, then
$$nsin(2pi n! e)=nsin(2pi I_n+2pi n!r_n)=nsin(2pi n!r_n).$$
Note
$$frac 1{n+1}le n!r_n=frac 1{n+1}+frac 1{(n+1)(n+2)}+cdotslesum_{k=1}^inftyfrac1{(n+1)^k}=frac 1n.$$
By sandwich theorem, $n!r_nto 0$ and $ncdot n!r_nto 1$ as $ntoinfty$. Using $sinthetasimtheta$ as $thetato 0$, we get
$$lim_{ntoinfty}nsin(2pi n! e)=lim_{ntoinfty}nsin(2pi n! r_n)=2pi.$$





To prove $e$ irrational, assume $e=p/q$, then when $n>q$, $n!einBbb Z$ implies $sin(2pi n! e)equiv 0$, and
$$lim_{ntoinfty}nsin(2pi n! e)=lim_{ntoinfty}ncdot 0=0ne 2pi,$$
a contradiction.






share|cite|improve this answer














Let $I_n=sumlimits_{k=0}^ndfrac{n!}{k!}in Bbb Z$, then
$$nsin(2pi n! e)=nsin(2pi I_n+2pi n!r_n)=nsin(2pi n!r_n).$$
Note
$$frac 1{n+1}le n!r_n=frac 1{n+1}+frac 1{(n+1)(n+2)}+cdotslesum_{k=1}^inftyfrac1{(n+1)^k}=frac 1n.$$
By sandwich theorem, $n!r_nto 0$ and $ncdot n!r_nto 1$ as $ntoinfty$. Using $sinthetasimtheta$ as $thetato 0$, we get
$$lim_{ntoinfty}nsin(2pi n! e)=lim_{ntoinfty}nsin(2pi n! r_n)=2pi.$$





To prove $e$ irrational, assume $e=p/q$, then when $n>q$, $n!einBbb Z$ implies $sin(2pi n! e)equiv 0$, and
$$lim_{ntoinfty}nsin(2pi n! e)=lim_{ntoinfty}ncdot 0=0ne 2pi,$$
a contradiction.







share|cite|improve this answer














share|cite|improve this answer



share|cite|improve this answer








edited Nov 18 at 13:12

























answered Nov 18 at 13:03









Tianlalu

2,654632




2,654632












  • I love you :), you made it clear, thanks!
    – Lamethyste
    Nov 18 at 13:07












  • The key idea here is the bound for $r_n$. If $e$ were rational say $m/n$ then $n! e=I_n+n! r_n$ is an integer and this is a contradiction as $n! r_nin[1/(n+1),1/n] $.
    – Paramanand Singh
    Nov 18 at 14:07




















  • I love you :), you made it clear, thanks!
    – Lamethyste
    Nov 18 at 13:07












  • The key idea here is the bound for $r_n$. If $e$ were rational say $m/n$ then $n! e=I_n+n! r_n$ is an integer and this is a contradiction as $n! r_nin[1/(n+1),1/n] $.
    – Paramanand Singh
    Nov 18 at 14:07


















I love you :), you made it clear, thanks!
– Lamethyste
Nov 18 at 13:07






I love you :), you made it clear, thanks!
– Lamethyste
Nov 18 at 13:07














The key idea here is the bound for $r_n$. If $e$ were rational say $m/n$ then $n! e=I_n+n! r_n$ is an integer and this is a contradiction as $n! r_nin[1/(n+1),1/n] $.
– Paramanand Singh
Nov 18 at 14:07






The key idea here is the bound for $r_n$. If $e$ were rational say $m/n$ then $n! e=I_n+n! r_n$ is an integer and this is a contradiction as $n! r_nin[1/(n+1),1/n] $.
– Paramanand Singh
Nov 18 at 14:07




















 

draft saved


draft discarded



















































 


draft saved


draft discarded














StackExchange.ready(
function () {
StackExchange.openid.initPostLogin('.new-post-login', 'https%3a%2f%2fmath.stackexchange.com%2fquestions%2f3003449%2ftrying-to-prove-es-irrationality%23new-answer', 'question_page');
}
);

Post as a guest















Required, but never shown





















































Required, but never shown














Required, but never shown












Required, but never shown







Required, but never shown

































Required, but never shown














Required, but never shown












Required, but never shown







Required, but never shown







Popular posts from this blog

Le Mesnil-Réaume

Ida-Boy-Ed-Garten

web3.py web3.isConnected() returns false always